Tuesday, February 23, 2010

HOMEWORK ASSIGNMENT 4 SOLUTIONS

BIOLOGY1361
HOMEWORK ASSIGNMENT 4 SOLUTIONS

Review Exercises / Critical Thinking problems

Chapter 23: (1 – 23) on page 662

1. Explain what occurs between the time of ovulation and the implantation of the fertilized in the uterus. After ovulation, the ovum enters the abdominal cavity and then finds its way into the uterine tubes. Fertilization of the ovum usually occurs in the outer one third of the uterine tubes; the fertilized egg is then called a zygote. After about 3 days of constant cell division, a solid mass of cells, called the morula, is formed. The cells of the morula continue to divide by the time the developing embryo reaches the uterus, it is a hollow ball of cells called a blastocyst. At this time, implantation into the uterine wall occurs.
2. Explain the function of the chorion and placenta.
a. Chorion – develops into an important fetal membrane in the placenta. The chorionic villi connect the blood vessels of the chorion to the placenta.
b. Placenta – The placenta anchors the developing fetus to the uterus and provides a bridge for the exchange of nutrients and waste products between the mother and baby.
3. Name the three primary germ layers, and name three structures that develop from each layer.
a. One - Endoderm
b. Two - Mesoderm
c. Three – Ectoderm
(there are too many to list for each)
4. Define histogenesis and organogenesis.
a. Histogenesis – development of the primary germ layers into the various tissues of the body.
b. Organogenesis – development of the various tissues into organs
5. Describe and give the approximate length of each of the three stages of labor.
a. Stage One – onset of uterine contractions until dilation of the uterus is complete, from 6 hours to 24 hours;
b. Stage Two – this begins at maximum cervical dilation and lasts until the baby exits through the vagina, usually from a few minutes to an hour
c. Stage Three – begins at birth of the baby and ends when the placenta is expelled through the vagina; this lasts about 15 minutes after birth of the baby.
6. What is the difference between identical and fraternal twins?
a. Identical twins have identical DNA
b. Fraternal twins are no closer in DNA than any two siblings
7. What is an ectopic pregnancy? Where is it most like to occur? This happens when a zygote is implanted outside the uterus. The most common type of ectopic occurs in the uterine tube. This is called tubal pregnancy.
8. What is placenta previa. What is abruptio placentae?
a. Placenta previa – this occurs when the blastocyst implants in the uterine wall too near the cervical opening of the uterus
b. Abruptio placentae – This happens when the placenta separate from the uterine wall after pregnancy of more than 20 weeks.
9. What’s preeclampsia? – A disorder characterized by the onset of acute hypertension after the 24th week of pregnancy, accompanied by proteinuria and edema. It can result in complication such as abruptio placentae, stroke, hemorrhage, fetal malnutrition, and low birth weight.
10. What is a teratogen? An agent that can disrupt the normal histogenesis and organogenesis, causing a congenital abnormality in the fetus.
11. What is the stimulus for a baby’s first breath? An increase in carbon dioxide concentration in blood cells after the umbilical is terminated.
12. Name three developmental changes that occur during infancy.
a. One – birth weight doubles during first 4 months, and triples in first year
b. Two – Length increases by 50% first year
c. Three – Baby fat decreases
d. And others
13. Briefly explain what biological developments occur during childhood. Rapid growth, less chubby, face matures, baby starts smoking cigars
14. Briefly explain what biological developments occur during adolescence. Rapid physical growth; sexual maturity; secretion of sex hormones; and stuff like that.
15. Briefly explain what biological developments occur during adulthood. Maturation of bone, full closure of plates, balding patterns, halitosis, and farts
16. What is progeria? – A genetic disorder causing advance aging in children.
17. Explain the effects of aging on the skeletal system. Bones undergo changes in texture and strength
18. Explain the effects of aging on the respiratory system. Harder cartilage, especially around rib cage
19. Explain the effects of aging on the cardiovascular system. Degenerative heart and blood vessel disease.
20. Explain the effects of aging on vision. Progressive myopia may give way to glaucoma and such.
21. Where do the nutrients used by the zygote between the time of fertilization and implantation come from? Egg cell.
22. Explain the evolution of the function of the yolk sac. Mostly to make blood cells in homo sapien sapien.
23. What hormones are produced by the placenta? What is their function? Chorionic gonadotropins, estrogen, progesterone, and they help maintain the uterine wall during pregnancy.

Chapter 24: (1-17) on page 684

1. Explain how the DNA code is able to regulate the biochemistry of the cell. Each gene is a sequence of nucleotide bases in the DNA molecule, containing a genetic code that the cell transcribes to a messenger RNA molecule. Each messenger RNA molecule moves to a ribosome where the code is translated into a specific protein molecule. Many of the protein molecules are enzymes that permit specific biochemical reactions to occur.
2. As they are used in this chapter, define chromosome and gene. The first is DNA, and the second is a code in the DNA.
3. What is meant by independent assortment? As the sperm and egg are formed, chromosome pairs are separate; the original maternal and paternal chromosomes get mixed up and re-distribute the sleeve independently of the other chromosome pairs. According each sperm and egg is like to have a different test of 23 chromosomes. This process I called independent assortment of chromosomes, and it ensures that each offspring will very likely be genetically unique.
4. Define or explain the terms dominant, recessive, and codominant in regards to genetics.
a. Dominant – a gene that is able to express itself and mask the presence of a recessive train
b. Recessive – a gene that is only able to express itself in the absence of a dominant gene
c. Co-dominant – gens that will express themselves differently in the respect of co-dominant genes
5. What is a sex-linked trait? Nonsexual traits that are carried on the sex-determining chromosomes. Because most are carried on the X chromosome, they are also referred to as X-lined traits.
6. Define or explain the terms nondisjunction, trisomy, and monosomy.
a. Nondisjunction – when a pair of chromosomes fails to separate
b. Trisomy – A triplet of a chromosome pair in a cell rather than the usual two.
c. Monosomy – there is a single chromosome of a chromosome pair rather than the usual two.
7. What’s a pedigree chart? A family tree
8. What is a Punnett square? A grid used to determine the probability of an inherited and genetic trait.
9. What is a karyotype? – Photograph of chromosomes that are cut and pasted on a chart. What are the two methods used to harvest cells for a karyotype? The cells can be harvested for karyotype by scraping the inside of the cheek, or by taking a sample of blood containing white blood cells. The cells can be harvested from a fetus by amniocentesis or villus sampling.
10. Explain the difference between gene augmentation and gene replacement therapy. In gene augmentation therapy, normal genes are introduced in to the body with the hope that the genes will add to or augment the production of a needed protein. In gene replacement therapy, the genes that specify production of abnormal , disease-causing proteins are replaced by normal or therapeutic genes.
11. Name and briefly describe the two single-gene diseases discussed in this chapter. Cystic fibrosis is caused by recessive genes in chromosomes pair 7. The primary effect is the impairment of chloride ion transport across cell membranes. The disruption causes exocrine cells to secrete thick mucus and swat. The thickened mucus is especially trouble in the g.i. and respiratory tract where it can cause obstruction. Phylketornuir is causes b the recessive genets that fail to produced the enzyme phelalanine hydroxilase. This enzyme is needed to convert the amino acid phenylalanine into thyroxide. Phylalanine absorbed from the ingested food accumulates, resulting in the abnormal presence of phylketone in the urine. A high concentration of phelalanine destroys brain tissue; babies born with this condition are at risk for progressive mental retardation and possibly death.
12. Name and briefly describe the three chromosomal diseases discussed in this chapter. Indicate whether the diseases are the result of the trisomy or monosomy.
a. One – Down Syndrome
b. Two - Klinefelter Syndrome xxy
c. Three - Turner Syndrome xo
13. Explain why the particular sperm that fertilizes the egg determine the sex of the offspring. The female as the sex chromosomes of xx; the male has xy; all of the eggs produced by the male have x chromosomes. The permp permitted by the male will have either X or Y. If the sperm with X fertilizes the eggs, a female offspring will be produce. If the sperm with a y chromosome fertilizes the egg, a male offspring will be produced.
14. What is crossing-over? How does this process add to generic variation?
During one phase of mitosis, pairs of matching chromosomes line of up along the equator of the cell and exchange genes. This process is called crossing over; genes from a particular location over to the same location of the matching genes. Sometimes a whole group stays together and crosses over as a single unit. Crossing over changes the linkage patterns on individual chromosomes and thereby increases the genetic variation among offspring of a set of parents.
15. Explain why men have more six-linked disorders than women. Many sex linked disorders in men are recessive Because women have two X chromosome, both chromosomes would have to carry the trait to be expressed. Because men have only one x chromosome, if that chromosome carries the train it will be express because there is no corresponding gene on the Y chromosome. Therefore, in men, even recessive trains on the X chromosome act if they were dominant traits.
16. Which type of genetic mutation has the greatest long-term impact on the population? Harmful or beneficial? Explain your answer. Beneficial mutations allow organization to adapt to their environment; therefore they tend to spread throughout a population over the course of several generations. Harmful mutations inhibit survival and are not as likely to spread through the population.
17. If parents are concern that their child might be born with Down syndrome, what would be the best way to determine this: a pedigree, a Punnett square, or karyotype? Explain your answer. A karyotype would be the best method of determining whether a child has Down syndrome. Down Syndrome is a chromosome disorder (trisoly21) A karyotype looks at chromosomes so that the presence of an extra chromosome if the 21st pair would determined

END

LESSON 11

Bio 1361 LESSON 11
TUESDAY, February 23, 2010
Dr Dave Menke, Instructor

I Logistics
A. Return Papers
B. Running Grades
C. Gather overdue papers
D. Questions

II ESSAY 4 FOR THIS WEEK: Nicolaes Tulp (1593-1674), due Thursday, 2/25

III Review of Lesson 10 – Comments on Growth

IV HOMEWORK ASSIGNMENT 4: Read Chapters* 23, 24 and do the Review Exercises / Critical Thinking problems (1 – 23) on page 662 and (1-17) on page 684. Turn in Thursday, 2/25

V LESSON 11: Review Chapters 13 – 24 to Prepare for Final Exam, for Rest of the Period

*From Textbook

Monday, February 22, 2010

LESSON 10

Bio 1361 LESSON 10
MONDAY, February 22, 2010
Dr Dave Menke, Instructor

I Logistics
A. Return Papers
B. Running Grades
C. Gather overdue papers
D. Questions

II ESSAY 4 FOR THIS WEEK: Nicolaes Tulp (1593-1674), due Thursday, 2/25

III Review of Lesson 9 - Chapters 23, 24: Growth

IV HOMEWORK ASSIGNMENT 4: Read Chapters* 23, 24 and do the Review Exercises / Critical Thinking problems (1 – 23) on page 662 and (1-17) on page 684. Turn in Thursday, 2/25

V LESSON 10: Review Chapters 1 – 12 to Prepare for Final Exam, for Rest of the Period

*From Textbook

Sunday, February 21, 2010

HOMEWORK ASSIGNMENT 4

BIOLOGY1361
HOMEWORK ASSIGNMENT 4

Review Exercises / Critical Thinking problems

Chapter 23: (1 – 23) on page 662

1. Explain what occurs between the time of ovulation and the implantation of the fertilized in the uterus.
2. Explain the function of the chorion and placenta.
3. Name the three primary germ layers, and name three structures that develop from each layer.
4. Define histogenesis and organogenesis.
5. Describe and give the approximate length of each of the three stages of labor.
6. What is the difference between identical and fraternal twins?
7. What is an ectopic pregnancy? Where is it most like to occur?
8. What is placenta previa. What ia abruptio placentae?
9. What’s preeclampsia?
10. What is a teratogen?
11. What is the stiumulus for a baby’s first breath?
12. Name three developmental changes that occur during infancy.
13. Briefly explain what biological developments occur during childhood.
14. Briefly explain what biological developments occur during adolescence.
15. Briefly explain what biological developments occur during adulthood.
16. What is progeria?
17. Explain the effects of again on the skeletal system.
18. Explain the effects of again on the respiratory system.
19. Explain the effects of again on the cardiovascular system.
20. Explain the effects of again on vision.
21. Where do the nutrients used by the zygote between the time of fertilization and implantation come from?
22. Explain the evolution of the function of the yolk sac.
23. What hormones are produced by the placenta? What is their function?


Chapter 24: (1-17) on page 684

1. Explain how the DNA code is able to regulate the biochemistry of the cell.
2. As they are used in this chapter, define chromosome and gene.
3. What is meant by independent assortment?
4. Define or explain the terms dominant, recessive, and codominant in regards to genetics.
5. What is a sex-linked trait?
6. Define or explain the terms nondisjunction, trisomy, and monosomy.
7. What’s a pedigree chart?
8. What is a Punnett square?
9. What is a karyotype? What are the two methods used to harvest cells for a karyotype?
10. Explain the difference between gene augmentation and gene replacement therapy.
11. Name and briefly describe the two single-gene diseases discussed in this chapter.
12. Name and briefly describe the three chromosomal diseases discussed in this chapter. Indicate whether the diseases are the result of the trisomy or monosomy.
13. Explain why the particular sperm that fertilizes the egg determine she sex of the offspring.
14. What is crossing-over? How does this process add to generic variation?
15. Explain why men have more six-linked disorders than women.
16. Which type of genetic mutation has the greatest long-term impact on the population? Harmful or beneficial? Explain your answer.
17. If parents are concern that their child might be born with Down syndrome, what would be the best way to determine this: a pedigree, a Punnett square, or karyotype? Explain your answer.

END

Friday, February 19, 2010

LESSON 9

Bio 1361 LESSON 9
THURSDAY, February 18, 2010
Dr Dave Menke, Instructor

I Logistics
A. Turn in Homework 3
B. Turn in Essay 3
C. Turn in any other past work
D. Running Grades
E. Questions

II Test 3 on Chapters 15-22

III Return of Papers (Labs, Essays, Homework 3, etc.)

IV Mind Game 3 (Crossword Puzzle)

V Review of Lesson 8. Chapters 20, 21, 22:
Fluids, Electrolytes, Reproduction

VI LESSON 9. Chapters 23, 24: Growth

VII HOMEWORK ASSIGNMENT 4:
Read Chapters* 23, 24 and do the Review Exercises / Critical Thinking problems (1 – 23) on page 662 and (1-17) on page 684. Turn in Thursday, 2/25

IX ESSAY 4 FOR NEXT WEEK.
Nicolaes Tulp (1593-1674), due Thursday, 2/25

X READ, ANSWER QUESTIONS ON HOMEWORK ASSIGNMENT 4 UNTIL END OF PERIOD

*From Textbook

Wednesday, February 17, 2010

HOMEWORK 3 SOLUTIONS

BIOLOGY1361
HOMEWORK ASSIGNMENT 3 SOLUTIONS

Review Exercises / Critical Thinking problems

Chapter 15: (1 – 24) on page 447

1. Define lymph and explain its function. It is a fluid found between the cells of the human body. It enters the lymph vessels by filtration through pores in the walls of capillaries. The lymph then travels to at least one lymph node before emptying ultimately into the right or the left subclavian vein, where it mixes back with blood.
2. Name the two major lymphatic ducts and the areas of the body each of them drains.
a. Right – right upper extremity, head, neck, etc.
b. Thoracic – the remaining ¾ of body
3. Describe the structure of a lymph node. From pinhead to lima bean in size with one or more vessels entering or leaving
4. What is lymphedema? What is the cause of elephantiasis?
a. Lymphedema is a condition of localized fluid retention and tissue swelling caused by a compromised lymphatic system
b. Elephantiasis is a disease that is characterized by the thickening of the skin and underlying tissues, especially in the legs, male genitals and female breasts. In some cases, the disease can cause certain body parts, such as the scrotum, to swell to the size of a softball or even a basketball.
5. Explain the defense function of the lymph node. Filter bacteria and other abnormal stuff, including dirt, bad cells, etc.
6. Where is the thymus gland? What are its functions?
a. Where – is a specialized organ of the lymphoid immune system in the mediastinum in the midline of the neck.
b. Functions - The only known function of the thymus is the production of T lymphocytes (T cells), which are critical cells of the adaptive immune system. The thymus is composed of two identical lobes and is located anatomically in the anterior superior mediastinumin front of the heart and behind the sternum
7. Name the three pairs of tonsils and give the location of each.
a. Tubal – aka pharyngeal, near the back opening of the nasal cavity
b. Palatine – on each side of the throat
c. Lingual – at the base of the tongue
8. Give the location and function of the spleen. High in the upper quadrant of the abdomen, lateral to the stomach. It removes many bacteria and bad things. It also destroys worn out red blood cells while saving the iron.
9. Explain the types of nonspecific immunity. It attacks any irritant or abnormal substance that threatens the environment. This includes, skin, mucous, mucous membranes, and tears.
10. Name and differentiate the four types of specific immunity.
a. Natural active – body gets sick and develops immunity
b. Natural passive – borrowed immunity (like a baby from its mom)
c. Artificial active – vaccination with dead or weak virus
d. Artificial passive – serum (borrowed from another person)
11. What are antibodies? What are antigens?
a. Antibodies – funny shaped protein
b. Antigens – unique shaped compound that fits “like a glove” to antibody
12. Explain the role of complement in the immune system. Like a mole or secret agent system, this is a highly specific pairs of antibody-antigens that seek out and destroy bad thingies.
13. Explain the role of the macrophage in the immune system. – Wandering killer cells that hunt out and slay evil invaders
14. Explain the development and function of B cells. First of all, B cells are above average, but not genius cells, like A cells. J.K. B cells form in the liver and bone marrow before birth, but only in the marrow as adults, and make antibodies
15. Explain the development and function of T cells. T stands for “thymus” and that’s where they’re made. They make poisons to cause the death and destruction of foreign spies.
16. What is an allergy? This is a hypersensitivity of the immune system to an outside irritant.
17. What is autoimmunity? Give an example of an autoimmune disease. An overly weird response to self antigens, such as lupus.
18. What is iso-immunity? Gave an example of an iso-immunity disorder. An overly weird response to external antigens, such as in organ transplants.
19. What are HLAs? How are they related to tissue typing? HLA stands for “Human leukocyte antigen”, which are antigens found on tissue cells of the body.
20. What is SCID? What is its cause? “Severe Combined Immune Deficiency,” caused by a congenital deficiency.
21. List three causes of acquired immunodeficiency syndrome.
a. One – bad eating habits
b. Two - drugs
c. Three – disease or trauma
22. Differentiate between lymphatic capillaries and blood capillaries. Explain how the different structure related to their function.
a. Lymphatic – tiny blind ended vessels distributed in tissue
b. Blood – tiny vessels that connect arterioles and venules.
23. Explain the role of lymph node in the spread of cancer. As this system picks up bad thingies and brings them to the nodes, they can carry cancer cells hither and yon.
24. Explain the difference in mechanism in the development of the allergic reaction of runny nose and hives, and the allergic reaction to poison ivy. Well, um, er, runny noses and such are caused by an immediate allergic reaction to something. However, a runny nose response is from B cells, while a poison ivy response is a delayed reaction due to T cells.

Chapter 16: (1-26) on pages 483-484

1. Differentiate between the respiratory membrane and the respiratory mucosa.
a. Respiratory membrane separates the alveoli from the blood
b. Respiratory mucosa is a mucous membrane that lines the respiratory tubes
2. List the function of the paranasal sinus. First, to drain nasal cavities; second, to make the skull lighter (less bone in the head); third, to resonate sounds we make.
3. What is the function of the auditory tube? It allows air pressure equilibrium in the nasal region
4. What is the function of the epiglottis? A cartilage trap door to prevent food and drink from going into the lungs
5. Describe, in decrease order of size, the air tubes of the lung. Trachea, primary bronchi, secondary bronchi
6. What constitutes an upper respiratory infection? An infection of the upper respiratory region, like ear, nose, throat, etc.
7. Describe rhinitis, pharyngitis, and laryngitis.
a. Rhinitis – infection of the nose area
b. Pharyngitis – of the pharynx
c. Laryngitis – of the larynx
8. What is IRDS? What substance is missing from the lunch that causes IRDS? “Infant respiratory distress syndrome” is a breathing problem for newborns, especially if they are very small or premature.
9. Describe the pleura What is the function of the pleural fluid? Pleura cover the outer surface of the lungs and the surface of the ribs to act as a lubricant to decrease friction between the two
10. What is atelectasis? It’s bad. A collapsed lung.
11. Describe Cheyne-Stokes respiration. Not to be confused with the Cheney-Biden disagreements, this instead is a stop-go breathing pattern caused by injury, disease, heart attack, or drug o.d.
12. Differentiate between external, internal, and cellular respiration. The first is outside the cell between lungs and blood; the second is between blood and cells; the third is when the cells use the oxygen in combustion to create energy.
13. Explain the mechanical process of inspiration. In this course, it means when one breathes in, and what happens to the lungs and muscles around there.
14. Explain the mechanical process of expiration. – Exhaling and stuff.
15. Explain how gas is exchanged between the lungs and the blood, and the blood and the tissues. First, by diffusion; second, by diffusion. Confusion?
16. How is oxygen carried in the blood? On the backs of camels. JK. By the hemoglobin
17. Name and explain the volumes that make up the vital capacity. Tidal volume – the normal air that ebbs and flows; expiratory reserve is the extra air that one can exhale when forcing oneself to do so; inspiratory reserve – forced extra air intake, e.g. “take a deep breath,” and residual volume is what’s left over in one’s lungs after one exhales as much as possible.
18. Explain the function of chemoreceptors in regulating breathing. When there is too much carbon dioxide, too little oxygen, and/or too much acid in the blood, they send messages to the respiratory system to fix it.
19. Explain the function of stretch receptors in regulating breathing. These prevent one from inhaling too much and exploding the lungs.
20. What is bronchitis? Inflammation of the bronchi
21. Distinguish between lobar pneumonia, bronchopneumonia, and aspiration pneumonia.
a. Lobar – the entire lung
b. Broncho – scattered portions of the bronchial system
c. Aspiration – due to inhaling dust and shit
22. How is tuberculosis transmitted from person to person? What is the pathogen that causes TB? One can inhale a sneeze of an infected person, or eat/swallow the TB bacillus drops, from the mycobacterium tuberculosis.
23. What process in emphysema causes the reduction in lung surface area? Air gets trapped in alveoli, they expand, pop, and fuse with nearby tissue, thus decreasing the lung surface able to breathe in air
24. What occurs to restrict breathing in asthma? Spasms of the smooth muscle in the walls of the bronchial air passages.
25. Explain the effect smoking has on the body’s ability to move material trapped in the respiratory mucosa. Smoke paralyzes the cilia used to move the mucous along in its cleaning job, resulting in coughing. So, stop smoking.
26. Explain the role of other systems in the regulation of respiration. Everything in the body must work together to make sure that the lungs are okay.

Chapter 17: (1-26) on pages 521-522

1. Name and describe the 4 layers of the wall of the gastrointestinal (g.i.) tract.
a. One – mucosa, mucous membrane of the esophogus
b. Two – submucosa, tissue layer below the above
c. Three – sub-submucosa, ha ha, I mean muscularis, 2 layers of muscle tissue that contract to move food along
d. Four – serosa, outermost covering of the g.i. tract
2. What’s the function of the uvula and soft palate?
a. Uvula – prevents food from entering nasal cavities
b. Soft palate – also prevents food from entering nasal cavities
3. Explain the function of the different types of teeth.
a. Incisors, to rip and tear
b. Molars, to crush and pulverize
4. Describe the 3 main parts of the tooth.
a. One – Crown
b. Two – Neck
c. Three - Root
5. What is leukoplakia? What could possibly develop from that? White patches ini the mouth that may become tumors; often found in the mouths of smokers
6. Distinguish between gingivitis and periodontitis. Gingivitis is inflammation of the gums, while periodontitis is the inflammation of the root of the tooth near the bone
7. Name the 3 pairs of salivary glands and describe where the duct from each enters the mouth.
a. One – Parotid, below and in front of each ear
b. Two – Submandibular, either side of the tongue’s “frenulum”
c. Three – Sublingual, floor of the mouth
8. What is the function of the upper and lower esophageal sphincter muscles? Beats me. How about the cardiac and pyloric ones? Cardiac one keeps the swallowed food down, the other sends the food along into the intestine
9. Define peristalsis. Okay, I will. Wavelike muscles contraction of the muscularis layer that move food through the g.i. tract.
10. What are the 3 parts of the triple therapy used to treat ulcers?
a. One – bismuth subsalicylate (e.g., Pepto-Bismol)
b. Two – tetracycline (anti-biotic)
c. Three – metronidazole or chlarithromycin (more anti-biotic)
11. Explain how bile from the liver and gallbladder reaches the small intestine. What is the function of cholecystokinin? Well, the liver and gallbladder have pipes or ducts that combine into one large duck (quack quack) dumping the bile into a cool tiny organ called the duodenum. Meanwhile, cholecystokinin is a hormone that stimulates the contraction of the gallbladder to shove the bile into the duodenum
12. What is the relationship between body weight and formation of gall stones? Fat people have more cholesterol, which leads to creation of gallstones.
13. What is hepatitis? What are the signs and symptoms of hepatitis?
Hepatitis is inflammation of the liver, with jaundice, liver enlargement, anorexia, abdominal discomfort, gray-white feces, dark urine.
14. What is contained in pancreatic juice? Enzymes that digest all types of foods
15. What do the bacteria in the large intestine contribute to the body? The by-products of these nice bacteria are vitamin K and Vitamin B
16. List the 7 subdivisions of new Pulte Homes. Just kidding. The 7 subdivisions of the large intestine: cecum, ascending colon, transverse colon, descending colon, sigmoid colon, rectum, and anal canal.
17. Describe the mesentery and the greater omentum. Not momentum, that’s in physics.
a. Mesentery – extension between the parietal and visceral layers of the peritoneum
b. Omentum – a pouch-like extension of the visceral peritoneum
18. What’s peritonitis? What’s ascites? Watup?
a. Peritonitis – inflammation of the peritoneum
b. Ascites – is an abnormal accumulation of the fluid in the peritoneal area
19. Differentiate between mechanical digestion and chemical digestion.
a. Mechanical – breaks food into tiny particles
b. Chemical – dissolves other stuff
20. Briefly describe the process of carbohydrate digestion. It starts in the mouth, then continues along the way
21. Briefly describe the process of fat digestion. This happens in the small intestine and such.
22. Briefly describe the process of protein digestion. Begins in the stomach
23. Explain the process of absorption. After being digested, it is absorbed through the mucous membrane of the intestines
24. What structures in the small intestine increase the internal surface area? What advantage is gained by this increase in surface area? There are folds in the lining of the mucosa, among other things; more surface area means more and fasting absorption
25. Bile doesn’t cause a chemical change; what’s the effect of bile on fat, and why does this make fat digestion more efficient? Bile acts like dishwashing liquid to break up the fat into smaller drops.
26. Some people are lactose intolerant. (And some are work-intolerant. But this isn’t about one of my sons). This means that they are less able to fully digest lactose sugar. Due to a lack of the enzyme lactase; milk products should be avoided or dealt with.

Chapter 18: (1-25) on page 539

1. Define anabolism and catabolism. Are they related to cannibalism?
a. Anabolism is the chemical reaction that builds complex chemical compounds from food molecules
b. Catabolism is the process that releases energy from food molecules
c. Cannibalism is no fun
2. Explain the function of the liver. To maintain normal body glucose concentration, and aids in the digestion of fats and things
3. Briefly explain the process of glycolysis. First step in carb digestion
4. Briefly explain the citric acid cycle. Second step in carb digestion
5. What is the function of the electron transport system? The energy released is in the form of high energy electrons and this creates ATP (Adenosine Tri Phosphate)
6. Explain the ways in which energy stored in ATP is different from energy stored in food molecules. The ATP is released quickly, as the other is done slowly.
7. List the hormones that tend to increase the amount of sugar in the blood: Growth, hydrocortisone, epinephrine, and glucagon.
8. When does fat catabolism usually occur? When the body is low on glucose to metabolize.
9. When does protein catabolism usually occur? When the body runs out of carbs and fats to metabolize.
10. Explain what is meant by a nonessential amino acid. While we need these, they are made by the body itself from the raw materials at hand.
11. Name 3 water-soluble and 3 fat-soluble vitamins. Here are some: thiamine, riboflavin, niacin, pantothenic acid, pyridoxine, B vitamins, biotin, folic acid, Vitamin C, etc.
12. What is avitaminosis? Name a disorder caused by avitaminosis. What vitamin deficiency (VD) causes this disorder?
a. Avitaminoisis is a deficiency of some vitamin
b. A disorder can be scurvy
c. The deficient vitamin is for scurvy is C
13. What are the signs and symptoms of Vitamin A hypervitaminosis? Hmm, how about, dry skin, hair loss, anorexia, barfing; then beyond that, headaches, mental bizarre-ness, enlarged liver, and so 4th.
14. Name 3 minerals needed by the body. Take your pick: calcium, chlorine, cobalt, copper, iodine, iron, magnesium, manganese, phosphorus, potassium, sodium, zinc. Although chlorine is a gas, not a mineral.
15. What is the function of vitamins and minerals in the body? Helps enzymes work properly, and also keeps nerves in tip top shape
16. Differentiate between basal and total metabolic rate. The first is the rate at which food is catabolized and the second is the rate at which the body burns energy in one day.
17. Distinguish between marasmus and kwashiorkor. Say what? You’ve got to be kidding. Weird names. Marasmus is a lack of proteins and calories overall; kwash-whatever is caused when there are enough calories, but not enough proteins in the diet.
18. Name and explain 3 ways heat can be lost through the skin. Conduction (touch), convection (movement), radiation (infrared light)
19. What is the cause of malignant hyperthermia? The abnormal increase in body temperature when exposed to anesthetics is a genetic disorder.
20. Distinguish between heat exhaustion and heatstroke in terms of a person’s body temperature. The first one is a warning signal, but the body can maintain proper core temperature. The second is when “all hell breaks loose” and one can’t control one’s temperature.
21. Differentiate between absorption and assimilation. The first is the movement of food from the digestive system into the circulation; the second is when the food molecules cross into the cells.
22. Explain the advantage the body gains by having the blood go through the hepatic portal system. The newly energized blood is run through the liver to ensure it’s healthy status.
23. Diagram the ATP-ADP cycle. Include where the energy is added and where the energy is released. (See some diagram somewhere)
24. A man went on a 10-day vacation. Whoop-dee-doo. His total metabolic rate was 2,600 Calories per day. (Note that the word “calorie” is capitalized because a food Calorie is really the same as 1,000 calories, with a small “c.”) His total calorie intake was 3300 calories a day. He started his vacation at 178 pounds (81 kilograms). What did he weigh when he got back from his holiday? Hint: 1.0 pound is the same as 3,500 extra calories per day. 180 pounds. Woo Hoo.
25. Why is trying to lower a moderate fever counterproductive to the body’s attempt to fight off an infection? Well, the body increases its temp to burn invaders, so don’t lower it unless it’s necessary.

Chapters 19 - 22 in Progress. Check back later.


Chapter 19: (1-23) on page 565

1. Describe the location of the kidneys – Just above the waistline
2. Name and describe the internal structure of the kidneys – Each kidney is far more complex that YOU can imagine! There are 6 sections:
a. Cortex
b. Medulla
c. Pyramids
d. Papilla
e. Pelvis
f. Calyx
3. Define filtration, reabsorption, and secretion as they apply to kidney function.
a. Filtration – movement of water and dissolved substances out of the blood
b. Re-absorption – movement of water and dissolved substances out of the kidneys
c. Secretion – movement of ions and ammonia and some chemicals (some drugs) out of the blood into the urine
4. Briefly explain the formation of urine. Sure. Blood flows through the glomeruli creates pressure high enough to push water into the Bowman’s capsule. Water, glucose, other nutrients, sodium, and other ions are re-absorbed in an endless loop, along with stuff from other organs are dumped into the bladder.
5. Name several substances eliminated or regulated by the kidney. Water, glucose, ions, and stuff like that.
6. Explain the function of the juxtaglomerular apparatus. This is a device that holds motorcycles for maintenance. JK. The jux thingy regulates blood pressure by releasing hormones that, well, regulate blood pressure.
7. Describe the structure of the ureters. Narrow tubes less than 6 millimeters wide and 30 cm long that bring fluids into the bladder.
8. Chemical urinalysis provides information about what substances in the urine? pH, urea, and the abnormal presence of other things
9. Casts are sometimes found in a urine sample; what are casts? Small particles formed by deposits or minerals on the walls of the renal tubules.
10. Describe the structure of the bladder. What is the trigone? Elastic fibers and involuntary muscles with a mucous membrane. A trigone is a triangular area at the “back” of the bladder.
11. Describe the structure of the urethra. Lowest point of the urinary tract, lined with a mucous membrane, about 4 cm long in women, 20 cm long in men.
12. Briefly explain the process of micturition. Two sphincter muscles allow the ebb and flow of urination (micturition) so one doesn’t just keep leaking urine, and also to prevent urine from escaping while the man is ejaculating semen.
13. Differentiate between retention and suppression of urine. The first one is when NO pee comes out; the second is when the kidneys don’t got no urine to send to the bladder.
14. What is incontinence? What can cause it? When a person urinates spontaneously without planning or wanting to, it may be due to illness or injury, or due to the aging of the sphincter muscles.
15. What is the most common urinary disorder? What can remove urine stains and odor? Bladder infection.
16. What is hydronephrosis? Not a good thing; urine backs up into the kidney, like an earthquake making the Mississippi River back up to Minnesota.
17. What is another term for renal calculi? What are they usually made of? Kidney stones, which are made of calcium, uric acid, or both.
18. Briefly explain the following disorders: urethritis, cystitis, and pyelonephritis.
a. Urethritis – inflammation of the urethra
b. Cystitis – inflammation of the bladder
c. Pyelonephritis – inflammation of the renal pelvis and connective tissue of the kidney
19. What is proteinuria? What is hypoalbuminemia?
a. Proteinuria – protein in the urine
b. Hypoalbuminemia – Low concentration of albumin in the blood
20. Briefly describe the 3 stages of chronic renal failure.
a. One – Lost nephrons (kidney cells) are replaced by existing cells that get bigger
b. Two – Lost nephrons can’t be covered any more
c. Three – The Blood Urea Nitrogen level goes sky high
21. Explain the salt and water balance maintained by aldosterone and ADH. Well, ADH decreased the amount of urine, but if no ADH is present, the tubes are impermeable to water; with ADH, the water is re-absorbed. Aldosterone controls the re-absorption of salt.
22. What is proper blood pressure necessary for proper kidney function? To as to push the fluids through the kidneys to be filtered.
23. If a person were doing strenuous work on a hot day and perspiring heavily, would there be a great deal of ADH in the blood, or very little? Explain. Vast amounts, so as to retain water.
24. There is no #24.

Chapter 20: (1-15) on page 583

1. Name and give the location of the 3 main fluid compartments of the body. Which of these make up extracellular fluid? (Intracellular, extracellular, and bladder, or the following):
a. Intracellular – in and among cells
b. Interstitial – around cells
c. Plasma – Fluid of the blood itself
2. What factors influence the percentage of water in the body? Explain the effect of each factor.
a. One – body weight
b. Two – Fat content
c. Three – Gender
d. Four - Age
3. List the 3 sources of water for the body. Tap water, bottled water, and restaurants. JK. Liquids we drink, food we eat, and as a by-product of biochemical reactions in our bodies.
4. List the 4 organs from which fluid output occurs. Kidneys, lungs, skin, intestines
5. Differentiate between an electrolyte and a non-electrolyte. An electrical light and an election. Jk. Electrolyte is ionic. The other ain’t.
6. Name 3 important negative ions. Chloride, bicarbonate, and phosphate. I think that there is a town in northern Arizona named Chloride.
7. Name 3 important positive ions. Sodium, Calcium, Potassium.
8. Explain why the body is unable to reduce its fluid output to zero no matter how dehydrated it is. Tissue and all cells have water in them, so they keep dehydrating until mummified or embalmed.
9. Explain how aldosterone influences water movement between the kidney tubules and the blood. Explain how Al Capone became a mob boss. Jk. We did this already. It increases the re-absorption of sodium
10. Explain the role of capillary blood pressure in water movement between the plasma and interstitial fluid. Another similar question, but it pushes the water along.
11. Explain the role of plasma proteins in water movement between the plasma and interstitial fluid. It’s a system that pulls or holds water.
12. Define dehydration and give a possible cause. Dehydration, which occurs when a body has less water than it needs to have, can come about due to fasting, excessive vomiting, dysentery, and disease.
13. Define over-hydration and give a possible cause. This is having too much fluid I the body. Not really possible naturally, but only if some stupid nurse sticks an I.V. into your vein, then squeezes the bag so the fluid zooms into your body really fast.
14. Name the 3 hormones that regulate urine volume. Sing “Urine the Army Now.” Jk. State where each is made and the specific effect each has on urine volume.
a. One – Aldosterone
b. Two – Antidiuretic hormone (ADH)
c. Three – Atrial natriuretic hormone (ANH)
15. Atrial natriuretic hormone has the opposite effect of aldosterone. Explain its effect on water movement between the kidney tubules and the blood. ANH causes sodium to be secreted into the urine.

Chapter 21: (1-14) on page 599

1. Explain the relationship between pH and the relative concentration of hydrogen and hydroxide ions in a solution. High pH number means low concentrations of hydrogen ions [H]+ and high concentration of hydroxide ions [OH]-.
2. Write out the chemical reaction formula that converts carbon dioxide and water to carbonic acid. Carbon dioxide dissolved in water is in equilibrium with carbonic acid:
CO2 + H2O ⇌ H2CO3
What enzyme acts as a catalyst for this reaction? Red blood cells contain carbonic anhydrase which increases the reaction rate
3. What is a buffer? An aqueous solution (water solution) consisting of a mixture of a weak acid and its conjugate base (or a weak base). The pH of solutions changes very little when a small amount of strong acid or base is added to it. Buffer solutions are used as a means of keeping pH at a nearly constant value in a wide variety of chemical applications. Many life forms thrive only in a relatively small pH range; an example of a buffer solution is blood. Who is Buffer, the Vampire Slayer? Jk
4. Explain how a buffer pair would react if more hydrogen ions were added to the blood. Limit change in pH.
5. Explain how a buffer pair would react if more hydroxide ions were added to the blood. Limit change in pH.
6. List the 4 changes that occur in the blood as the result of buffering fixed acids.
First of all, life in mammals are limited to a pH range between 6.8 and 7.8. Beyond those limits we end up with cellular failure.
7. Explain the respiratory mechanism of pH control. Oxygen is a prime part of water and hydroxide. Carbon dioxide is sent out of the body via the lungs. If pH changes, this loops adjusts.
8. Describe how changes in the respiration rate can affect blood pH, and how it can return the pH to optimum levels. If one slows breathing, there will be a buildup of carbon dioxide in the blood, which then affects the carbonic acid concentration, thus lowering the pH and setting into motion higher breathing rates. Plus, artery blood has a higher pH than venal blood.
9. Explain how the chemical reaction that occurs in the distal tubule of the kidney using NaH2PO4 (monosodium phosphate) to remove hydrogen ions from the blood. The kidneys remove excess acid from the blood. One way is to change Na2HPO4 (disodium phosphate) into NaH2PO4 (monosodium phosphate) which then goes to the bladder.
10. Define acidosis and alkalosis.
a. Acidosis – Acidosis is said to occur when arterial pH falls below 7.35
b. Alkalosis – Alkalosis is said to occur when arterial pH is over 7.45
11. Explain metabolic disturbances of the buffer pair. This is the amount of bicarbonate in the pair, leading to acidosis.
12. Explain respiratory disturbances of the buffer pair. This is the amount of carbonic acid in the pair, leading to alkalosis.
13. Explain how excessive vomiting causes metabolic alkalosis and explain why normal saline can be used to correct it. First of all, vomiting tosses lots of HCl, hydrochloric acid, out of the body, causing bells and whistles to go off. Addition of NaCl, salt, puts the chlorine balance back.
14. What is the proper ratio of NaHCO3 (sodium bicarbonate) and H2CO3 (carbonic acid) in a buffer pair? 20:1. Explain how the body can use this ratio to correct uncompensated metabolic acidosis. It sets off more bells and sirens, and causes one to breathe faster.

Chapter 22: (1-24) on page 634

1. Describe structure and location of the testes (aka testicles). The testes are located in the scrotum, a small, flexible sac hanging outside the male body under the penis. Each testicle (aka testis) is an oval gland – not a round one like a ball – 3.8 cm wide. A tough membrane surrounds each one.
2. Describe the structure of the spermatozoa. (Note that this word has a similar ending as the word protozoa). Head, mid-piece, and tail. Like a tadpole.
3. List the functions of testosterone. This hormone causes a boy to evolve into a man, increases muscle strength, and increases libido.
4. List and briefly describe the reproductive ducts (not ducks) of the male reproductive system. The epididymis is a single, tightly wound coil about 6 meters long! It’s inside the scrotum. The ductus deferens, aka vas deferens, is the tube that allows the semen to leave the sac and enter the main part of the body on its trip. The ejaculatory duct is at the junction of the ductus deferens and the seminal vesicle, allowing semen to be dumped into the urethra.
5. List and briefly describe the glands of the male reproductive system. What does each gland contribute to the seminal fluid?
a. Seminal vesicles – pouch-like glands that donate 60% of the seminal fluid, rich in sugary fructose
b. Prostate gland – a doughnut shaped organ located below the bladder, donates about 30% of the fluid
c. Bulbourethral glands – these are pea-shaped glands that contribute only about 5% of the volume, but the fluid is a lubricant
6. Distinguish between infertility, sterility, and impotence (not importance).
a. Infertility – Low probability of producing an offspring
b. Sterility – No probability of producing an offspring
c. Impotence – Inability to achieve an erect penis
7. What is ogliospermia? What is cryptochidism?
a. Ogliospermia – decrease in sperm production
b. Cryptochidism – when the testicles haven’t dropped into the scrotum
8. Both a hydrocele and an inguinal hernia will produce swelling in the scrotum (the sac where the testicles are). Explain the difference between the two.
a. Hydrocele – too much fluid in the scrotum
b. inguinal hernia – intestines rip through their membranes and interfere
9. Describe the structure and location of the ovaries. The ovaries resemble large almonds and are on either side of the uterus.
10. Explain the development of an ovarian follicle from the primary follicle to the corpus luteum. With a beginning of 1 million oocytes, they’ve decreased to about 400,000 by onset of menses, of which only 400 will mature to a true follicle.
11. List the functions of estrogen. This hormone causes development and maturation of a girl into a woman.
12. List the functions of progesterone. Initiates menstrual cycles.
13. Describe the structure of the uterine tubes. Ducts for ovaries.
14. Describe the structure of the uterus. Like a strong pear-shaped thingy.
15. Describe the structure of the vagina. A 10-cm tube of smooth muscle
16. Describe the structure of the breast. Each is a layer of fat on top of the pectoral muscles with 15 or 20 divisions of various gland lobes
17. Explain what occurs during the proliferative phase of the reproductive cycle. This is the period between end of menstrual cycle and ovulation.
18. Explain what occurs during the secretory phase of the reproductive cycle. This starts with ovulation and lasts until menstruation begins anew.
19. Name the 4 hormones involved in the regulation of the reproductive cycle. Where is each made, and what is the function of each?
a. One – Follicle stimulating hormone (FSH), anterior pituitary gland, causes growth of several follicles that then secrete estrogen
b. Two – Luteinizing hormone (LH), anterior pituitary gland , matures ovum and creates more estrogen
c. Three – Estrogen, made in ovary, creates growth and lining of uterus
d. Four – Progresterone, made in ovary, starts the menstrual cycle
20. What is dysmenorrhea? What is amenorrhea?
a. Dysmenorrheal – painful menstruation
b. Amenorrhea – abnormal menstruation
21. Distinguish between salpingitis and oophoritis.
a. Salpingitis – inflammation of uterine tubes
b. Oophoritis – inflammation of the ovaries
22. What is endometriosis? When this stuff grows outside the uterus
23. Differentiate between spermatogenesis and oogenesis. How do these differences relate to the role of the male and female in reproduction?
a. Spermatogenesis – the creation of spermatozoa, and millions need to be made to ensure pregnancy
b. Oogenesis – the creation of eggs or oocytes; only a few hundred need to be made to ensure pregnancy
24. Why are the testicles located outside the body cavity in the scrotum? It’s too hot for the haploid spermatozoa to survive.

END

Monday, February 15, 2010

LESSON 7

Bio 1361 LESSON 7
MONDAY, February 15, 2010
Dr Dave Menke, Instructor

I Logistics
A. Return Papers
B. Gather overdue papers
C. Running Grades
D. Questions
E. Lupercalia

II ESSAY 3 FOR THIS WEEK
Henry Gray (1827–1861), due Thursday, 2/18

III Review of Lesson 6 - Chapters 15, 16:
A. Respiration – Lungs breathe air
78% N2 (inert gas), 21% O2 (gas), Ar (gas), CO2 (gas)
B. Digestion
C. Lymph

IV Lab 4: Stuff dealing with chapters 15-22

V LESSON 7. Chapters 17, 18, 19: Digestion, Nutrition, and Urinary Systems

VI HOMEWORK ASSIGNMENT 3:
Read Chapters* 15-22 and do the Review Exercises / Critical Thinking problems (1 – 24) on page 447, (1-26) on pages 483-484, (1-26) on pages 521-522, (1-25) on page 539, (1-23) on page 565, (1-15) on page583, (1-14) on page 599, and (1-24) on page 634.Turn in Thursday, 2/18

VII READ, ANSWER QUESTIONS ON HOMEWORK ASSIGNMENT 3 UNTIL END OF PERIOD


*From Textbook

Dr Dave’s Delicious Recipes

Dr Dave’s Delicious Recipes
By Dr Dave Menke
DrDHMenke@aol.com

1. Dr Dave’s Delicious Pan Fried Tilapia

a. Ingredients
4 Tilapia Filets
Olive oil
Italian Bread Crumbs
Butter

b. Directions
Warm frying pan with olive oil, until hot
Put tilapia in pan, pour some oil on each
When one side is done, flip
Add some butter to pan for flavor
Pour bread crumbs over tilapia, lightly
Flip, pour bread crumbs
Remove and serve

Side dishes: macaroni & cheese; fresh steamed asparagus

Serves 4

2. Dr Dave’s Delicious Chicken Paprikash

a. Ingredients
4 large chicken breasts (boneless, skinless)
Olive oil
Paprika
Small onion, diced
Chicken Broth
Sour Cream

b. Directions
Sauté onion in olive oil, add paprika
When onion is soft, add chicken
Brown chicken on both sides, then put in a pot
Pour onion and paprika in the frying pan over the chicken in the pot
Pour One can of Broth over the chicken (or more)
Simmer 30 minutes til tender
Remove from heat, add cup of sour cream, stir
Let sit 5 minutes, then serve

Side dishes: thin sliced cucumber salad with oil & vinegar; garlic spaghetti

Serves 4

3. Dr Dave’s Delicious Honey Mustard Chicken

a. Ingredients
4 large breasts (boneless, skinless)
Olive oil
Honey – 6 TBS
Mustard – 2 TBS

b. Directions
Pre-heat oven to 350° F
Using Pyrex dish or similar, pour enough olive oil to cover bottom
Place chicken in baking dish
Pour honey, then mustard, over chicken
Baste and turn every 15 minutes for one hour
Serve

Side dishes: Cut up and boiled new potatoes, with butter and parsley; broccoli supreme*

Serves 4

*steam broccoli, then put in Pyrex dish, pour a mixture of shredded cheddar + cottage cheese + minced onions over top. Bake at 350 for 10 minutes.

4. Dr Dave’s Delicious Meat Loaf

a. Ingredients
One pound of ground beef (or veal or turkey)
Can of “Veg-all” mixed vegetables
2 eggs
Olive Oil
Cup shredded cheddar or similar
Minced onions, about ¼ cup
Ketchup
Mustard
Tobasco Sauce by McIlhenny
Worcestershire sauce
Salt and pepper to taste
Italian Bread crumbs

b. Directions
Pre heat oven to 350° F
Beat 2 eggs
Add meat
Add all other ingredients and mix thoroughly
Place in oiled Pyrex dish for one hour, with foil on top
Take out and let stand 5 minutes
Serve, with hot rolls or bread

Serves 4

5. Dr Dave’s Delicious Chicken Kiev

a. Ingredients
Chicken breasts - 4
Bread Crumbs
Butter (or Margarine)
Oven
Pyrex dish
Aluminum Foil
Spices
Olive Oil

b. Directions
1. Pre-Heat Oven to 375° F
2. Pour olive oil into Pyrex dish, and make sure it covers all the bottom part of the dish
3. Start with room temperature chicken breast(s). They need to feel like a limp dick. Cut off any fat
4. Wash chicken and pat dry
5. Flatten the Breasts as much as possible
6. Cut a stick of butter into quarters, length-wise
7. Place the ¼ stick of butter on the flat side of the chicken breast
8. Fold the breast over the butter
9. Hold the breast-butter together with fingers (or use tootpicks, etc), while rolling it in bread crumbs (Italian spiced bread crumbs or similar is recommended; or use plain bread crumbs and add these spices: salt, pepper, paprika, chili powder, basil, Mrs Dash for chicken, oregano, and anything that sounds interesting; mix all the spices with the bread crumbs, then roll the chicken in it)
10. Place the chicken breast, closed side down, in the Pyrex dish (that means, the two parts held together by one’s fingers should be face down on the glass of the dish)
11. Sprinkle chicken with remaining bread crumbs that are left over
12. Cover chicken with Aluminum Foil
13. Place dish in oven for 40 minutes*
14. Remove foil, continue cooking for 20 minutes
15. Take Chicken Kiev from Oven, let stand 2 or 3 minutes
16. Serve. The Pyrex dish will have a buttery-olive oil sauce mixed with bread crumbs, so spoon some of this over each serving for exquisite taste. (NOTE: cutting into warm chicken breast may cause the buttery inside to squirt, so be prepared)

*This time is good for regular, fat type breasts. If breasts are thinner, less time is required, so test for done-ness.

Feeds 4

6. Dr Dave’s Delicious Chicken (or other) Fried Rice

a. Ingredients
Rice-a-Roni Fried Rice – 1 package
Chicken – breast or thigh; or pork; or beef
Eggs – 2
Sweet Peas – small can
Olive Oil

b. Directions
Prepare Rice-a-Roni Fried Rice per package
Scramble and cook two eggs
Grill chicken or pork or beef, then dice
Add eggs to Fried Rice when Rice is done
Add peas to Rice when Rice is done
Add meat to Rice when it’s done

Serves 2 – 3

7. Dr Dave’s Delicious Beef & Broccoli in Oyster Sauce

a. Ingredients
Sirloin steak,  pound
Broccoli Florets – one bunch (fresh or frozen)
Oyster Sauce – small bottle

b. Directions
Grill steak until done, then cut into small pieces
Steam broccoli until done
Combine steak and broccoli
Pour half of a small bottle of Oyster Sauce into beef-broccoli, and mix
Drain so just a coating of Sauce is on Beef and Broccoli
Serve

Serves 2

8. Dr Dave’s Delicious Broccoli Supreme
Ingredients:
Broccoli florets (fresh or frozen) - 1 lb. (I like fresh)
Shredded Cheddar Cheese - 2 cups (I prefer Mexican 4 cheese shredded)
2 eggs
Cottage Cheese – one cup (regular)
Minced Onions – 1 tablespoon; or small raw onion, chopped
Seasoned Bread crumbs (homemade or packaged), 1 cup (I like Italian seasoning)
Cream of Celery Soup – half can
Olive Oil – half cup
Instructions:
1. Steam broccoli, drain, set aside
2. Spray inside of baking dish / pyrex dish with Olive Oil (like Pam, etc.)
3. Arrange steamed broccoli in baking dish.
4. Beat eggs, add to soup, cottage cheese, and onions and mix; pour over broccoli
5. Sprinkle with cheese
6. Combine olive oil and bread crumbs, mix, then sprinkle on top
7. Bake at 350 degrees for 30 minutes.
Makes 4 side servings with meal

Any questions? Call I-800-EAT-SHIT. (just kidding)

HOMEWORK ASSIGNMENT 3 COMPLETE

BIOLOGY1361
HOMEWORK ASSIGNMENT 3

Review Exercises / Critical Thinking problems

Chapter 15: (1 – 24) on page 447

1. Define lymph and explain its function.
2. Name the two major lymphatic ducts and the areas of the body each of them drains.
3. Describe the structure of a lymph node.
4. What is lymphedema? What is the cause of elephantiasis?
5. Explain the defense function of the lymph node.
6. Where is the thymus gland? What are its functions?
7. Name the three pairs of tonsils and give the location of each.
8. Give the location and function of the spleen.
9. Explain the types of nonspecific immunity.
10. Name and differentiate the four types of specific immunity.
11. What are antibodies? What are antigens?
12. Explain the role of complement in the immune system.
13. Explain the role of the macrophage in the immune system.
14. Explain the development and function of B cells.
15. Explain the development and function of T cells.
16. What is an allergy?
17. What is autoimmunity? Give an example of an autoimmune disease.
18. What is iso-immunity? Gave an example of an iso-immunity disorder.
19. What are HLAs? How are they related to tissue typing?
20. What is SCID? What is its cause?
21. List three causes of acquired immunodeficiency syndrome.
22. Differentiate between lymphatic capillaries and blood capillaries. Explain how the different structure related to their function.
23. Explain the role of lymph node in the spread of cancer.
24. Explain the difference in mechanism in the development of the allergic reaction of runny nose and hives, and the allergic reaction to poison ivy.

Chapter 16: (1-26) on pages 483-484

1. Differentiate between the respiratory membrane and thee respiratory mucosa.
2. List the function of the paranasal sinus.
3. What is the function of the auditory tube?
4. What is the function of the epiglottis?
5. Describe, in decrease order of size, the air tubes of the lung.
6. What constitutes an upper respiratory infection?
7. Describe rhinitis, pharyingitis, and laryngitis.
8. What is IRDS? What substance is missing from the lunch that causes IRDS?
9. Describe the pleura What is the function of the pleural fluid?
10. What is atelectasis?
11. Describe Cheyne-Stokes respiration.
12. Differentiate between external, internal, and cellular respiration.
13. Explain the mechanical process of inspiration.
14. Explain the mechanical process of expiration.
15. Explain how gas is exchanged between the lung and the blood, and the blood and the tissues.
16. How is oxygen carried in the blood?
17. Name and explain the volumes that make up the vital capacity.
18. Explain the function of chemoreceptors in regulating breathing.
19. Explain the function of stretch receptors in regulating breathing.
20. What is bronchitis?
21. Distinguish between lobar pneumonia, bronchopneumonia, and aspiration pneumonia.
22. How is tuberculosis transmitted from person to person? What is the pathogen that causes TB?
23. What process in emphysema causes the reduction in lung surface area?
24. What occurs to restrict breathing in asthma?
25. Explain the effect smoking has on the body’s ability to move material trapped in the respiratory mucosa.
26. Explain the role of other systems in the regulation of respiration.


Chapter 17: (1-26) on pages 521-522

1. Name and describe the 4 layers of the wall of the gastrointestinal (g.i.) tract.
2. What’s the function of the uvula and soft palate?
3. Explain the function of the different types of teeth.
4. Describe the 3main parts of the tooth.
5. What is leukoplakia? What could possibly develop from that?
6. Distinguish between gingivitis and periodontitis.
7. Name the 3 pairs of salivary glands and describe where the duct from each enters the mouth.
8. What is the function of the upper and lower esophageal sphincter muscles?
9. Define peristalsis.
10. What are the 3 parts of the triple therapy used to treat ulcers?
11. Explain how bile from the liver and gallbladder reaches the small intestine. What is the function of cholecystokinin?
12. What is the relationship between body weight and formation of gall stones?
13. What is hepatitis? What are the signs and symptoms of hepatitis?
14. What is contained in pancreatic juice?
15. What do the bacteria in the large intestine contribute to the body?
16. List the 7 subdivisions of new Pulte Homes. Just kidding. The 7 subdivisions of the large intestine.
17. Describe the mesentery and the greater omentum. Not momentum, that’s in physics.
18. What’s peritonitis? What’s ascites? Watup?
19. Differentiate between mechanical digestion and chemical digestion.
20. Briefly describe the process of carbohydrate digestion.
21. Briefly describe the process of fat digestion.
22. Briefly describe the process of protein digestion.
23. Explain the process of absorption.
24. What structures in the small intestine increase the internal surface area? What advantage is gained by this increase in surface area?
25. Bile doesn’t cause a chemical change; what’s the effect of bile on fat, and why does this make fat digestion more efficient?
26. Some people are lactose intolerant. (And some are work-intolerant. But this isn’t about one of my sons). This means that they are less able to fully digest lactose sugar. What enzyme is probably not functioning properly and what type of food should these people try to avoid?

Chapter 18: (1-25) on page 539

1. Define anabolism and catabolism. Are they related to cannibalism?
2. Explain the function of the liver.
3. Briefly explain the process of glycolysis.
4. Briefly explain the citric acid cycle.
5. What is the function of the electron transport system?
6. Explain the ways in which energy stored in ATP is different from energy stored in food molecules.
7. List the hormones that tend to increase the amount of sugar in the blood.
8. When does fat catabolism usually occur?
9. When does protein catabolism usually occur?
10. Explain what is meant by a nonessential amino acid.
11. Name 3 water-soluble and 3 fat-soluble vitamins.
12. What is avitaminosis? Name a disorder caused by avitaminosis. What vitamin deficiency (VD) causes this disorder?
13. What are the signs and symptoms of Vitamin A hypervitaminosis?
14. Name 3 minerals needed by the body.
15. What is the function of vitamins and minerals in the body?
16. Differentiate between basal and total metabolic rate.
17. Distinguish between marasmus and kwashiorkor. Say what?
18. Name and explain 3 ways heat can be lost through the skin.
19. What is the cause of malignant hyperthermia?
20. Distinguish between heat exhaustion and heatstroke in terms of a person’s body temperature.
21. Differentiate between absorption and assimilation.
22. Explain the advantage the body gains by having the blood go through the hepatic portal system.
23. Diagram the ATP-ADP cycle. Include where the energy is added and where the energy is released.
24. A man went on a 10-day vacation. Whoop-dee-doo. His total metabolic rate was 2,600 Calories per day. (Note that the word “calorie” is capitalized because a food Calorie is really the same as 1,000 calories, with a small “c.”) His total calorie intake was 3300 calories a day. He started his vacation at 178 pounds (81 kilograms). What did he weigh when he got back from his holiday? Hint: 1.0 pound is the same as 3,500 extra calories per day.
25. Why is trying to lower a moderate fever counterproductive to the body’s attempt to fight off an infection?

Chapter 19: (1-23) on page 565

1. Describe the location of the kidneys
2. Name and describe the internal structure of the kidneys
3. Define filtration, reabsorption, and secretion as they apply to kidney function.
4. Briefly explain the formation of urine.
5. Name several substances eliminated or regulated by the kidney.
6. Explain the function of the juxtaglomerular apparatus.
7. Describe the structure of the ureters.
8. Chemical urinalysis provides information about what substances in the urine?
9. Casts are sometimes found in a urine sample; what are casts?
10. Describe the structure of the bladder. What is the trigone?
11. Describe the structure of the urethra.
12. Briefly explain the process of micturition.
13. Differentiate between retention and suppression of urine.
14. What is incontinence? What can cause it?
15. What is the most common urinary disorder? What can remove urine stains and odor?
16. What is hydronephrosis?
17. What is another term for renal calculi? What are they usually made of?
18. Briefly explain the following disorders: urethritis, cystitis, and pyelonephritis.
19. What is proteinuria? What is hypoalbuminemia?
20. Briefly describe the 3 stages of chronic renal failure.
21. Explain the salt and water balance maintained by aldosterone and ADH.
22. What is proper blood pressure necessary for proper kidney function?
23. If a person were doing strenuous work on a hot day and perspiring heavily, would there be a great deal of ADH in the blood, or very little. Explain.
24. There is no #24.

Chapter 20: (1-15) on page 583

1. Name and give the location of the 3 main fluid compartments of the body. Which of these make up extracellular fluid?
2. What factors influence the percentage of water in the body? Explain the effect of each factor.
3. List the 3 sources of water for the body.
4. List the 4 organs from which fluid output occurs.
5. Differentiate between an electrolyte and a non-electrolyte. An electrical light and an election. Jk
6. Name 3 important negative ions.
7. Name 3 important positive ions.
8. Explain why the body is unable to reduce its fluid output to zero no matter how dehydrated it is?
9. Explain how aldosterone influences water movement between the kidney tubules and the blood. Explain how Al Capone became a mob boss. Jk
10. Explain the role of capillary blood pressure in water movement between the plasma and interstitial fluid.
11. Explain the role of plasma proteins in water movement between the plasma and interstitial fluid.
12. Define dehydration and give a possible cause.
13. Define over-hydration and give a possible cause.
14. Name the 3 hormones that regulate urine volume. Sing “Urine the Army Now.” Jk. State where each is made and the specific effect each has on urine volume.
15. Atrial natriuretic hormone has the opposite effect of aldosterone. Explain its effect on water movement between the kidney tubules and the blood.

Chapter 21: (1-14) on page 599

1. Explain the relationship between pH and the relative concentration of hydrogen and hydroxide ions in a solution.
2. Write out the chemical reaction formula that converts carbon dioxide and water to carbonic acid. What enzyme acts as a catalyst for this reaction?
3. What is a buffer? Who is Buffer, the Vampire Slayer? Jk
4. Explain how a buffer pair would react if more hydrogen ions were added to the blood.
5. Explain how a buffer pair would react if more hydroxide ions were added to the blood.
6. List the 4 changes that occur in the blood as the result of buffering fixed acids.
7. Explain the respiratory mechanism of pH control.
8. Describe how changes in the respiration rate can affect blood pH.
9. Explain how the chemical reaction that occurs in the distal tubule of the kidney using NaH2PO4 (monosodium phosphate) removed hydrogen ions from the blood.
10. Define acidosis and alkalosis.
11. Explain metabolic disturbances of the buffer pair.
12. Explain respiratory disturbances of the buffer pair.
13. Explain how excessive vomiting causes metabolic alkalosis and explain why normal saline can be used to correct it.
14. What is the proper ratio of NaHCO3 (sodium bicarbonate) and H2CO3 (carbonic acid) in a buffer pair? Explain how the body can use this ratio to correct uncompensated metabolic acidosis.

Chapter 22: (1-24) on page 634

1. Describe structure and location of the testes (aka testicles).
2. Describe the structure of the spermatozoa. (Note that this word has a similar ending as the word protozoa).
3. List the functions of testosterone.
4. List and briefly describe the reproductive ducts (not ducks) of the male reproductive system.
5. List and briefly describe the glands of the male reproductive system. What does each gland contribute to the seminal fluid?
6. Distinguish between infertility, sterility, and impotence (not importance).
7. What is ogliospermia? What is cryptochidism?
8. Both a hydrocele and an inguinal hernia will produce swelling in the scrotum (the sac where the testicles are). Explain the difference between the two.
9. Describe the structure and location of the ovaries.
10. Explain the development of an ovarian follicle from the primary follicle to the corpus luteum.
11. List the functions of estrogen
12. List the functions of progesterone
13. Describe the structure of the uterine tubes.
14. Describe the structure of the uterus.
15. Describe the structure of the vagina.
16. Describe the structure of the breast.
17. Explain what occurs during the proliferative phase of the reproductive cycle.
18. Explain what occurs during the secretory phase of the reproductive cycle.
19. Name the 4 hormones involved in the regulation of the reproductive cycle. Where is each made, and what is the function of each?
20. What is dysmenorrhea? What is amenorrhea?
21. Distinguish between salpingitis and oophoritis.
22. What is endometriosis?
23. Differentiate between spermatogenesis and oogenesis. How do these differences relate to the role of the male and female in reproduction?
24. Why are the testicles located outside the body cavity in the scrotum?

END

Sunday, February 14, 2010

LESSON 6

Bio 1361 LESSON 6
THURSDAY, February 11, 2010
Dr Dave Menke, Instructor

I Logistics
A. Turn in homework, essays, etc.
B. Running Grades
C. Questions

II Test 2

III Return of Papers (Lab 3, Essays, Homework 2, etc.)

IV Mind Game 2 (Story)

V Review of Lesson 5: Chapters 12, 13, 14: Blood, Heart, Circulation

VI LESSON 6. Chapters 15, 16: Respiration, Digestion, Lymph

VII HOMEWORK ASSIGNMENT 3:
Read Chapters* 15-22 and do the Review Exercises / Critical Thinking problems (1 – 24) on page 447, (1-26) on pages 483-484, (1-26) on pages 521-522, (1-25) on page 539, (1-23) on page 565, (1-15) on page583, (1-14) on page 599, and (1-24) on page 644.Turn in Thursday, 2/11

IX ESSAY 3 FOR NEXT WEEK.
Henry Gray (1827–1861), due Thursday, 2/18

X READ, ANSWER QUESTIONS ON HOMEWORK ASSIGNMENT 3 UNTIL END OF PERIOD

*From Textbook

Wednesday, February 10, 2010

HOMEWORK SOLUTION SET 2 (Completed at 10 PM)

BIOLOGY1361
HOMEWORK SOLUTION SET 2

Review Exercises / Critical Thinking problems

Chapter 7: (1 – 20) on page 203

1. List and describe briefly the 5 functions of the skeleton/skeletal system.
a. support
b. protection
c. manufacture of cells
d. movement
e. strength
2. What’s the “osteon”? The fundamental functional unit of compact or dense bone; in mammals, birds, reptiles
3. What’s the function of cartilage? Provide more flexibility at joints and to provide support where bone is not used. No blood vessels in cartilage.
4. What’s the process of endochondral ossification (include osteoblasts and osteoclasts)? One of two processes resulting in the formation of normal, healthy bone tissue, done in presence of cartilage, creates the bone tissue.
5. What is the purpose of the epiphyseal plate? A cartilage plate that turns into bone, present in kids, but not so much in adults.
6. What bones are in the axial skeleton? Skull, neck, spinal cord, rib cage; In the appendicular skeleton? The rest (limbs, etc.)
7. Name the 5 sections of the vertebral column with the number of vertebrae in each.
a. Cervical, 7
b. Thoracic, 12
c. Lumbar, 5
d. Sacral, 5
e. Coccygeal, 4
8. What the heck are true, false, and floating ribs? How many each are there?
a. True: The 7 true ones that are fully formed in the sternum
b. False: The false ribs are the five sets of ribs below the top 7 true ribs
c. Floating: Four ribs the lower end, attached to the vertebral column, not to the sternum
9. What’s a synarthrotic joint? A type that allows limited movement
10. What’s an amphiarthrotic joint? Two-way
11. What’s a diarthrotic joint? Same as synovial joint
12. What’s a joint capsule? an envelope surrounding a synovial joint
13. What are open, closed, and comminuted fractures?
a. Open – through the skin
b. Closed – not through skin
c. Comminuted – hair fracture
14. The three types of arthritis are:
a. Osteo
b. Rheumatoid
c. Gout
d. Many others
15. When a patient receives a bone marrow transplant, what vital process is being restored? Manufacture of blood cells
16. Explain how the canaliculi allow bone to heal more efficiently than cartilage. It supplies blood to the bone; no blood vessels in cartilage
17. What effect does the task of child bearing have on the difference between the male and female skeleton? Female has a wider pelvis to accommodate
18. Is it possible to tell whether a child is going to grow any taller? If so, how? If not, who cares? Yes, check growth hormone; genetics
19. Compare and contrast the causes and changes associated with osteoporosis, osteomalacia, and Paget disease. See cure in today’s article*
20. Why is mastoiditis potentially more dangerous than a paranasal sinus infection? mastoiditis is a deep ear infection that can spread to the brain and kill the person.

*Cure for Osteoporosis Has Been Found
HealthDay.com
February 8, 2010

An experimental drug that inhibits serotonin in the gut cured osteoporosis in lab mice and rats, a new study has found.
Previous research has shown that serotonin in the gut hinders bone formation. Most current drugs for osteoporosis only prevent the breakdown of bone; they don't build bone.
"New therapies that inhibit the production of serotonin in the gut have the potential to become a [new] class of drugs to be added to the therapeutic arsenal against osteoporosis," study author Dr. Gerard Karsenty, chairman of the department of genetics and development at Columbia University College of Physicians and Surgeons, said in a university news release.
In the new study, published in the Feb. 7 issue of Nature Medicine, small daily doses of the new drug were given by mouth for up to six weeks to rodents with postmenopausal osteoporosis. The treatment prevented osteoporosis from developing or fully cured rodents in which osteoporosis was already present.
While the drug inhibited serotonin in the gut, levels of serotonin remained normal in the rodents' brains. This indicates that the drug didn't enter general circulation and wasn't able to cross the blood-brain barrier, thus greatly reducing the risk of side effects.
"With tens of millions of people worldwide affected by this devastating and debilitating bone loss, there is an urgent need for new treatments that not only stop bone loss but also build new bone. Using these findings, we are working hard to develop this type of treatment for human patients," Karsenty said.

Chapter 8: (1-22) on pages 230-231

1. Describe the structure of the cardiac muscle. Involuntary striated
2. Describe the structure of the smooth muscle. Involuntary, not striated
3. Give the function of tendons, bursae, and synovial membranes.
a. Tendons – connects bones along and across bones
b. Bursae – fluid filled sacs that cushion bones and such
c. Synovial membranes - is the soft tissue that lines the non-cartilage surfaces of joints
4. Explain how tonic contractions help maintain posture. These are sustained contraction of a muscle subjected to vibration so as to hold one up
5. Give an example of how two body systems other than the muscular system contribute to the movement of the body.
a. One The movement of body structures is accomplished by the contraction of muscles
b. Two See (a)

6. Explain twitch and tetanic contractions. When a motor unit has been maximally stimulated by its neuron; This happens due to multiple impulses stimulating the motor unit and not giving it time to relax between contractions, thus the strength of impulse is increased as it is added to the previous impulse, creating a maximal contraction.
7. Explain isotonic contractions. The muscle tension remains unchanged and the muscle's length changes. Lifting an object off a desk is an example of isotonic contractions.
8. Explain isometric contractions. A type of strength training Isometrics are done in static positions, rather than being dynamic movement; weight lifting.
9. Name two muscles in the head, or neck and give the origin, insertion, and function of each.
a. One: Sternomastoid muscle - a paired muscle in the superficial layers of the anterior portion of the neck. It acts to flex and rotate the head. It also acts as an accessory muscle of inspiration.
b. Two: Scalene muscles - are a group of three pairs of muscles in the lateral neck, namely the scalenus anterior, medius, and posterior. They are innervated by the spinal nerves. They originate from the transverse processes from the cervical vertebrae of C2 to C7 and insert onto the first and second ribs. The function of the anterior and middle scalene muscles is to elevate the first rib and rotate the neck to the same side; the action of the posterior scalene is to elevate the second rib and tilt the neck to the opposite side. They also act as accessory muscles of inspiration,

10. Name two muscles that move the upper extremity and give the origin, insertion and function of each.
a. Trapezius; rear of skull, clavicle, retraction of scapula
b. Pectoralis major; clavicle & sternum, humerus, movement of humerus
11. Name two muscles of the trunk and give the origin, insertion and function of each.
a. Abdominal external oblique muscle; lower 8 costae, ligamentum iguinali, rotates torso
b. Abdominal internal oblique muscle; inguinal ligament, sternum & inferior ribs, compresses abdomen
12. Name three muscles of the lower extremity and give the origin, insertion and function of each.
a. Tibialis posterior; tibia, foot, dorsiflex & invert foot and ankle
b. Longus; fibula, foot, plantarflexion
c. Tibialis anterior; tibia, foot, dorsiflex & invert foot
13. Describe the following movements: flexion, extension, abduction, adduction, and rotation.
a. Flexion – a position that is made possible by the joint angle decreasing. The skeletal and muscular systems work together to move the joint into a "flexed" position.
b. Extension – movement of a joint whereby one part of the body is moved away from another
c. Abduction – a type of movement which draws a limb away from the median plane of the body
d. Adduction – a movement which brings a limb closer to the y-z plane of the body. It is opposed to abduction
e. Rotation – a movement that allows joint to move in rotation
14. What signs and symptoms are likely to accompany a moderate muscle strain?
localized pain, stiffness, discoloration, strange odors and bruising.
15. What causes the signs and symptoms of myasthenia gravis?
Fatigue; Muscles become progressively weaker during periods of activity and improve after periods of rest
16. Draw and label a relaxed sarcomere; explain the process that causes a sarcomere to contract. (a sarcomere is the basic unit of muscle tissue; multi-protein complexes composed of three different filament systems).
17. Explain the interaction of the prime mover, the synergist, and the antagonist efficient movement. The first is the main muscle, the second is a catalyst to help it work, and the antagonist is what the prime mover is working against
18. Describe the condition that causes a muscle to develop pan “oxygen debt.” How is the debt paid off? The muscle uses more oxygen than “normal” and runs low on it, sometimes stealing from other tissues, but it is paid off by the person breathing harder, faster for a period of time.
19. Why can a spinal cord injury be followed by muscle paralysis? The nerves that control the muscles can be “cut off” and no longer send signals.
20. Can a muscle contract very long if its blood supply is shut off? Give a reason for your answer. No, except in rigor mortis
21. Briefly explain changes that gradually take place in bones, joints, and muscles in a person who habitually gets too little exercise. Atrophy, weakening, calcification
22. Briefly explain the progression of Duchenne muscular dystrophy. Chronic and terminal withering of muscle tone and strength

Chapter 9: (1-31) on page 276

1. Draw and label the three parts of the neuron and explain the function of the dendrite and axon. A nerve cell is the neuron, with the dendrites being the branched projections that conduct the electrical impulses and the axon is the long slender “cable” that connects one cell to another.
2. Name the three types of neurons classified to the direction in which the impulse is being transmitted. Define or explain each of them.
a. Afferent neurons convey information from tissues and organs into the central nervous system and are sometimes also called sensory neurons
b. Efferent neurons transmit signals from the central nervous system to the effector cells and are sometimes called motor neurons
c. Interneurons connect neurons within specific regions of the central nervous system.
Afferent and efferent can also refer generally to neurons which, respectively, bring information to or send information from the brain region.
3. Define or explain the following terms: myelin, nodes of Ranvier, and neurolemma.
a. Myelin – An electrical insulator material that forms a layer, the myelin sheath, usually around only the axon of the neuron
b. Nodes of Ranvier - the gaps (approximately one micrometer in length) formed between the myelin sheaths generated by different cells.
c. Neurolemma - the outermost nucleated cytoplasmic layer of Schwann cells the surrounds the axon of the neuron. It forms the outermost layer of the nerve fiber in the peripheral nervous system.
4. Name and give the function of the 3 types of glia cells. First of all, they are called neuroglia or simply glia (Greek for "glue"), and are non-neuronal cells that maintain homeostasis, form myelin, and provide support and protection for the brain's neurons. In the human brain, there is roughly one glia for every neuron with a ratio of about two neurons for every three glia in the cerebral gray matter.
a. Microglia: these are like specialized macrophages capable of phagocytosis that protect neurons of the central nervous system. They are derived from hematopoietic precursors rather than ectodermal tissue; they are commonly categorized as such because of their supportive role to neurons.
These cells comprise approximately 15% of the total cells of the central nervous system. They are found in all regions of the brain and spinal cord. Microglial cells are small relative to macroglial cells, with changing shapes and oblong nuclei. They are mobile within the brain and multiply when the brain is damaged. In the healthy central nervous system, microglia processes constantly sample all aspects of their environment (neurons, macroglia and blood vessels).
b. Macroglia: There are 7 types of these cells; the most relevant are the astrocytes.
c. Other: See b.

5. What occurs at the cellular level in M.S.? And what effect does this have on the body? The myelin sheaths around the axons disintegrate; MS affects the ability of nerve cells in the brain and spinal cord to communicate with each other.
6. From what type of cells or tissues do neuromas develop? This is a tumor in neurons
7. Define or explain the following terms: epineurium, perineurium, and endoneurium.
a. Epineurium - the outermost layer of connective tissue surrounding a peripheral nerve. It includes the blood vessels supplying the nerve
b. Perineurium - The perineurium is composed of connective tissue, and acts as a protective sheath around the fascicles.
c. Endoneurium – A protective sheath of cells wrapped around nerve cells.
8. What causes gray matter to be gray, and white matter to be white? What color is the White House? In living tissue, gray matter actually has a gray-brown color which comes from capillary blood vessels and neuronal cell bodies; white matter is from the white myelin sheaths
9. Explain how a reflex arc functions. What are the two types of reflex arc?
a. How – It is the neural pathway that mediates a reflex
b. Type one – autonomic reflex arc (affecting inner organs)
c. Type Two - somatic reflex arc (affecting muscles)
10. Explain what happens during a nerve impulse. What is salutatory conduction?
a. What happens - a transient alteration of the transmembrane voltage (or membrane potential) across an excitable membrane in an excitable cell such as a neuron, generated by the activity of voltage-gated ion channels embedded in the membrane.
b. A saltatory conduction - is the propagation of action potentials along myelinated axons from one node of Ranvier to the next node, increasing the conduction velocity of action potentials without needing to increase the diameter of an axon
11. Explain what occurs at a synapse. What are the two ways that neurotransmitter activity is terminated?
a. What occurs – a structure that permits a neuron to pass an electrical or chemical signal to another cell.
b. One way – chemical
c. Another way - electrical
12. What is the cause of Parkinson Disease? What are some treatment options?
a. Cause – a degenerative disorder of the central nervous system that often impairs the sufferer's motor skills, speech, and other functions
b. Treatment – Medicine: L-Dopa and other meds
13. Define Dementia; unless you have dementia: Also known as "deprived of mind") it is a serious cognitive disorder. It may be the result of a unique global brain injury or progressive, resulting in long-term decline in brain function, due to damage or disease in the body beyond what might be expected from normal aging. Although dementia is far more common in old people, it may occur in any stage of adulthood. Dementia is a non-specific illness with its own set of signs and symptoms, in which affected areas of cognition may be memory, attention span, language skills, and being able to solve problems. It is normally required to be present for at least 6 months to be diagnosed
14. What is a seizure (not a legal seizure by the FBI)? A seizure is a transient symptom of excessive or synchronous neuronal activity in the brain. It can manifest as an alteration in mental states, gross motor movements, convulsions, and various other related things. The medical syndrome of recurrent, unprovoked seizures is termed epilepsy, but seizures can occur in people who do not have epilepsy.
15. List two possible causes of Alzheimer’s, if you can remember.
a. reduced synthesis of the neurotransmitter acetylcholine.
b. amyloid plaque deposits
16. List and describe the function of the medulla oblongata – This is the lower half of the brainstem, and contains the cardiac, respiratory, vomiting, and vasomotor centers and deals with autonomic functions, such as breathing, heart rate and blood pressure.
17. List and describe the function of the hypothalamus - a portion of the brain that contains a number of small nuclei with a variety of functions. One of the most important functions of the hypothalamus is to link the nervous system to the endocrine system via the pituitary gland.
18. List and describe the function of the thalamus - a midline paired symmetrical structure within the brain. It is situated between the cerebral cortex and midbrain, both in terms of location and neurological connections. Its function includes relaying sensation, special sense and motor signals to the cerebral cortex, along with the regulation of consciousness, sleep and alertness.
19. List and describe the functions of the cerebellum – this means “little brain” in Latin. Not to be confused by Sarah Belham. It is a region of the brain that plays an important role in motor control. It is also involved in some cognitive functions such as attention and language, and probably in some emotional functions such as regulating fear and pleasure responses, but its function in movement is the most clearly understood. The cerebellum does not initiate movement, but it contributes to coordination, precision, and accurate timing
20. Give the general function of the cerebrum. What are the specific functions of the occipital and temporal lobes?
a. Function of cerebrum – movement, sensory, olfaction, language, learning, memory
b. Functions of the occipital and temporal lobes – primary visual cortex and auditory cortex
21. List and describe the functions of the spinal cord - a long, thin, tubular bundle of nervous tissue and support cells that extends from the brain down to the space between the first and second lumbar vertebrae; it is the body’s sensor apparatus and the brain’s messenger service.
22. Name and explain the three layers of the meninges. This is the system of membranes which envelops the central nervous system. The primary function of the meninges is to protect the central nervous system.
a. Dura Mater - is a thick, durable membrane, closest to the skull
b. Arachnoid Mater - The middle element of the meninges with a spider-web appearance
c. Pia Mater - is a very delicate membrane. It is the meningeal envelope which firmly adheres to the surface of the brain and spinal cord.
23. What’s the function of the cerebrospinal fluid? Where and how is it produced?
a. What function: acts as a "cushion" or buffer for the cortex, providing a basic mechanical and immunological protection to the brain inside the skull.
b. Where and how: It is produced in the choroid plexus
24. How many nerve pairs are generated from the spinal cord? How many nerve pairs are generated from each section of the spinal cord? How are these nerves named? What is a “plexus”? What is a Lexus?
a. How many and How many from each section: The animal spinal cord is divided into 31 different segments. At every segment, right and left pairs of spinal nerves (mixed; sensory and motor) form. Six to eight motor nerve rootlets branch out of right and left ventro lateral sulci in a very orderly manner.
b. How are they named? I have no clue
c. Plexus is an area where neurons branch and rejoin to form a network
25. Define neuritis and neuralgia
a. Neuritis – also known as neuropathy, the term for damage to nerves of the peripheral nervous system. This may be caused either by diseases of the nerve or from the side effects of systemic illness.
b. Neuralgia – is a pain in one or more nerves that occurs without stimulation of pain receptor cells. Neuralgia pain is produced by a change in neurological structure or function rather than by the excitation of pain receptors that causes nociceptive pain.
26. What’s the cause of tic douloureux? What is the cause of Bell’s Palsy?
a. Tic douloureux – a neuropathic disorder of one or both of the trigeminal nerves. Symptoms include severe pain, and the fact that it is not easily controlled or cured. It causes episodes of intense pain in any or all of the following: the ear, eye, lips, nose, scalp, forehead, teeth or jaw on one side and alongside of the face.
b. Bell’s Palsy - a dysfunction of cranial nerve (the facial nerve) that results in inability to control facial muscles on the affected side. Several conditions can cause this. However, if no specific cause can be identified, the condition is known as Bell's palsy.
27. Explain the structure and function of the sympathetic nervous system.
a. Structure – Part of the autonomic nervous system
b. Function - mobilize the body's resources under stress; to induce the fight or flight. It is, however, constantly active at a basal level in order to maintain homeostasis.
28. Explain the structure and function of the parasympathetic nervous system.
a. Structure – Part of the autonomic nervous system
b. Function - control smooth muscle contraction, regulate cardiac muscle, or stimulate or inhibit glandular secretion. The actions of the parasympathetic nervous system can be summarized as "rest and digest
29. List the functional regions of the frontal, parietal, occipital, and temporal lobes.
a. Frontal – is located at the front of each cerebral hemisphere and positioned anterior to (in front of) the parietal lobes and above and anterior to the temporal lobes.
b. Parietal – is positioned above (superior to) the occipital lobe and behind (posterior to) the frontal lobe. The parietal lobe integrates sensory information from different things, particularly determining spatial sense and navigation.
c. Occipital – located on the medial side of the occipital lobe within the calcarine sulcus; the full extent of V1 often continues onto the posterior pole of the occipital lobe.
d. Temporal - is located beneath the Sylvian fissure on both the left and right hemispheres of the brain.
30. Which of the cranial nerves deal primarily with motor function? Which deal primarily with the sensory function?
a. Motor – Frontal Lobe
b. Sensory – Cerebrial cortex
31. There is a type of medication that inhibits the function of acetylcholinesterase (the enzyme that deactivates acetylcholine). Explain the effect this medication would have on the visceral effectors. Only one, neostigmine, affects vision.

Chapter 10: (1-25) on page 305

1. Name the general sense found in the skin or subcutaneous tissues and list the type of stimuli to which each of them responds. Tactile sensations of heat, cold, pleasure, pain.
2. Name the two general senses of proprioception and give the location of each.
This is a third distinct sensory modality that provides feedback solely on the status of the body internally, located in the cerebellum.
3. With what type of information do proprioceptors provide us? feedback solely on the status of the body
4. Explain how the iris changes the size of the pupil. It opens and closes as a reaction to light, as a camera iris.
5. Explain how the ciliary muscles allow the eye to focus on near and far objects. By squeezing the lens, it’s focal length changes.
6. What is presbyopia and what is its cause? This is far-sightedness causes by eyes that are short.
7. Name the two types of receptor cells in the retina. Explain the differences between the two receptors. Rods and cones. One has peripheral vision the other views colors.
8. What is glaucoma and what’s its cause? It is a disease in which the optic nerve is damaged, leading to progressive, irreversible loss of vision. It is often, but not always, associated with increased pressure of the fluid in the eye
9. What are cataracts, what causes them, and how can they be prevented?
Cataracts are cloudy lenses caused by the fluid in the lens becoming cloudy, and its causes include genetics and environment.
10. What is meant by the visual pathway. Where is the blind spot and what causes it? The path that light travels through the eye. The blind spot is where the optic nerve is in junction with the retina.
11. Explain how the disorder strabismus affects vision. Cross-eyed makes the person see two images.
12. What causes diabetic retinopathy? Diabetes.
13. Briefly explain the structure of the external ear. Crescent shape with many swirls ending at the eardrum.
14. Explain how sound waves are transmitted through the middle ear. Waves vibrate a bone which then is translated to the brain.
15. Explain how sound waves are converted to an auditory impulse. Electrically.
16. Explain how the structure in the inner ear help maintain balance or equilibrium. The fluid acts as a fluid balance like a carpenter’s balance
17. What is Méniere disease? It is a disorder of the inner ear that can affect hearing and balance to a varying degree. It is characterized by episodes of dizziness and tinnitus and progressive hearing loss, usually in one ear. It is caused by lymphatic channel dilation
18. Where are the gustatory cells located, and to what four “primary“ tastes do they respond. In the cerebrum, sweet, sour, salty, and bitter.
19. Explain how the sense of smell is stimulated. Through olfactory nerves that sense the concentration or density of items.
20. Explain why food loses some of its taste when you have a rhinovirus with a congested nasal passage. Taste buds are part of the olfactory system.
21. Explain why the longer you are in a newly painted room the less you are able to smell the new paint? The new paint molecules stimulate the olfactory until they are satiated.
22. Where in the eye is light sensed? Where is it perceived? Sensed at the retina and perceived in the brain.
23. Explain why the smell of a “doctor’s office” or the smell of a turkey cooking can easily generate an emotional response. Repeated smells associated with one place will create an “aromatic memory” and you will then remember those times
24. Why are there many more men, than women, color-blind? They’re liars. No, just kidding. The xy chromosomes have more effects in the color gene.
25. Rock musicians sometimes lose their ability to hear high frequency tones; explain. The ear’s ability to hear high frequencies is sensitive, and loud music destroys the most sensitive.


Chapter 11: (1-26) on pages 336-337

1. Differentiate between endocrine and exocrine glands. The endocrine system is made up of a series of ductless glands that produce chemicals called hormones. A number of glands that signal each other in sequence is usually referred to as an axis, for example, the hypothalamic-pituitary-adrenal axis. Typical endocrine glands are the pituitary, thyroid, and adrenal glands. Features of endocrine glands are, in general, their ductless nature, their vascularity, and usually the presence of intracellular vacuoles or granules storing their hormones. In contrast, exocrine glands, such as salivary glands, sweat glands, and g.i. glands, tend to be much less vascular and have ducts or a hollow lumen. Also controls metabolism in our body system.
2. Define or explain the following terms:
a. Hormone – a chemical released by one or more cells that affects cells in other parts of the organism
b. target organ – the organ to be affected
c. hypersecretion – makes and releases too much
d. hyposecretion – makes and releases too little
3. Explain the mechanism of action of non-steroid hormones. These are drugs with analgesic, antipyretic, (fever-reducing) and, in higher doses, with anti-inflammatory effects . The most prominent members of this group of drugs are aspirin, ibuprofen, and naproxen.
4. Explain the mechanism of action of steroid hormones. These are steroids that act like hormones.
5. Explain, and give an example of a negative feedback loop for the regulation of hormone secretion. Getting AIDS or some STD from allowing one’s hormones dictate behavior.
6. Explain, and give an example of a positive feedback loop for the regulation of hormone secretion. Having a baby .
7. Explain the difference between prostaglandins and hormones. List some of the body functions that can be influences by prostaglandins. These include any member of a group of lipid compounds that are derived enzymatically from fatty acids and have important functions in the body. They are mediators and have a variety of strong physiological effects, such as regulating the contraction and relaxation of smooth muscle tissue. Although they are technically hormones, they are rarely classified as such.
8. Describe the structure of the pituitary gland and where it is located. Not very big and at base of brain near spine.
9. Name the 4 tropic hormones released by the anterior pituitary gland and briefly explain their functions.
a. TSH – stimulate production of stuff
b. ACTH - stimulate production of other stuff
c. LH - stimulate production of more stuff
d. FSH - stimulate production of cool stuff
10. Explain the function of growth hormone. To ensure that you grow as tall as your ancestors or whatever your genome states.
11. Gigantism and acromegaly have the same cause; what is the cause and what causes the different in effect between the two conditions?
a. Cause – imbalance in hormone production
b. Difference – tall vs. large
12. Explain the function of ADH. This acts on kidneys to speed up process.
13. What is the cause of diabetes insipidus? What are the signs and symptoms of the condition?
a. Cause - a deficiency of arginine vasopressin (AVP), also known as ADH.
b. Signs/symptoms Excessive thirst for cold water and and excretion of large amounts of severely diluted urine, with reduction of fluid intake having no effect on the latter
14. Explain the function of prolactin and oxytocin.
a. Prolactin – stimulate production of milk in breast glands
b. Oxytocin – stimulate reduction of milk
15. Explain the function of the hypothalamus in the endrocrine system. It links the nervous system to the endrocrine system via the pituitary gland.
16. Explain the difference between T3 and T4. What’s unique about the thyroid gland? Tyrosine-based hormones produced by the thyroid gland primarily responsible for regulation of metabolism
17. Distinguish between cretinism and myxedma. Cretinism is a condition of severely stunted physical and mental growth due to untreated congenital deficiency of thyroid hormones due to maternal nutritional deficiency of iodine. While myxedma is hypothyroidism that affects skin and eyes.
18. Name the hormones produced by the zones or areas of the adrenal cortex. Androgens and corticoids.
19. What are the signs and symptoms of Cushing syndrome? Of Addington’s disease.
a. Cushing a hormone disorder caused by high levels of cortisol in the blood.
b. Addington’s a disorder where people can’t add
20. Explain the function of aldosterone - a hormone that increases the reabsorption of sodium and water and the release (secretion) of potassium in the kidneys. This increases blood volume and, therefore, increases blood pressure.
21. Explain the function of glucocorticoids – Steroid hormones that regulate of the metabolism of glucose
22. Explain why a secondary messenger system is needed for non-steroid hormones, but not for steroid hormones. Just in case.
23. Pick a body function (regulation of glucose or calcium levels in the blood) and explain how the interaction of hormones is used to help maintain homeostasis. Sex. One needs the right recipe of hormones to make sex cells correctly and the motivation to mate.
24. Why is a goiter usually more of a dietary problem rather than an endocrine problem? Causes overeating and lethargy
25. A medical doctor discovered that a patient had very low levels of thyroxine by noting high levels of TSH. Is the patient’s problem in the thyroid gland or in the pituitary gland? Explain your answer. Thyroid as the person is low on energy.
26. If a person who is diagnosed with diabetes mellitus were found to be producing a normal amount of insulin what other cause could explain the diabetes? Tumor.

Chapter 12: (1-19) on pages 369-370

1. Name several substances found in blood plasma. Blood cells, dissolved proteins, glucose, clotting factors, mineral ions, hormones, and carbon dioxide
2. Explain the function of albumins, globulins, and fibrinogen.
a. Albumins – water soluble protein
b. Globulins – serum protein
c. Fibrinogen – fibrous protein involved in the clotting of blood
3. What is the difference between serum and plasma? Serum is a protein.
4. What two types of connective tissue form blood cells? Where are they found and what do each of them form? Bone and the others.
5. Describe the structure of a red blood cell. What advantage does this unique shape give the red blood cell that helps it perform its function? Carries iron and oxygen.
6. Both aplastic anemia and pernicious anemia are characterized by low red blood cell count; explain the difference in their causes. Aplastic is from the bone marrow having a low production of red blood cells; pernicious is the loss of ability to absorb vitamin B-12
7. What is the buffy coat? Leukocyte and platelet layer between plasma and blood
8. Explain the function of neutrophils and monocytes; neutrophils are the most abundant type of white blood cell, while monocytes are specialized white blood cells that organize and move fast
9. Explain the function of lymphocytes. They kill bad bacteria
10. Explain the function of eosinophils and basophils. Eos- are white blood cells that fight parasites; baso- are the least number of white blood cells and deal with allergic reactions.
11. Distinguish the difference between leucopenia and leukocytosis. The decrease of white blood cells; above average white cells.
12. How is hemophilia transmitted? What blood clotting factors can be affected?
Genetically; missing chemicals.
13. Explain the process of blot clot formation. The hormone causes the plasma to become more like a glue.
14. Differentiate between a thrombus, embolus, and city bus. Thrombus is a swollen area. Embolus is a blod clot; city bus is transportation.
15. Explain how Type A blood differs from Type B blood. Different organization.
16. Explain the cause of erythroblastosis fetalis. (What a mouthful): malformation of organs in new born
17. Explain how heparin inhibits blood clot formation. Blocks hormones that cause it to clot.
18. Differentiate between the process of blood clot formation and the process of blood agglutination. One is to heal a wound, the other is to pool the blood.
19. Why is the first Rh-positive baby born to a mother of Rh-negative blood usually not affected? Not enough antibodies first delivery to harm the fetus.

Chapter 13: (1-17) on page 392

1. Describe the heart and its position in the body. The heart is a muscular blood pump, kept far enough above the ground for protection, and surrounded by bones for more protection.
2. Name the four chambers of the heart. Left, right atrium; left, right ventricle
3. What’s the myocardium? The endocardium? Striated heart muscle, interior heart muscles
4. Describe the two layers of the pericardium. Explain the function of the pericardial fluid; serous and fibrous; a double-walled sac that contains the heart and the roots of the great vessels.
5. Define or explain pericarditis and pericardial effusion. Inflammation of the outer layer of the heart, and its spread.
6. What’s systole? Diastole?; contraction (flex); release (extend)
7. Name and give the locations of the 4 heart valves. Inside
8. Explain what is meant by a mitral valve prolapsed. A heart disease of the valve characterized by the displacement of an abnormally thickened mitral valve into the left atrium during systole.
9. Explain what occurs in a myocardial infarction. Heart attack.
10. Trace the flow of blood from the superior vena cava to the aorta. In, around, out
11. What’s angina pectoris? False heart attack.
12. Differentiate between stroke volume and cardiac output. One is size, other is flow
13. Trace the path and name the structures involved in the conduction system of the heart. There are quite a few.
14. What is heart block? What’s bradycardia? What’s tachycardia. First is when one or more arteries are blocked, others are heart conditions.
15. What is fibrillation? Which is more dangerous, atrial fibrillation or ventricular fibrillation? When heart doesn’t contract, and ventricular is.
16. Explain how the tracing on an ECG relate to the electric activity of the heart. The heart is operated on electricity
17. Explain how right-sided heart failure is usually caused by left-sided heart failure. Bad input = bad output.

Chapter 14: (1-14) on page 418

1. Name and describe the main types of blood vessels in the body. Arteries, veins
2. Name the three tissue layers that make up arteries and veins.
3. What’s arteriosclerosis? Hardening of arteries that may lead to heart attack
4. What’s ischemia? Gangrene? First, ; second, blood infection
5. What’s an aneurism? Bursting vein
6. What’s phlebitis? Blood clot
7. Describe both systemic and pulmonary circulation. Lungs, lung system
8. Name and briefly explain the four factors that influence blood pressure. Ratio of sodium & potassium; weight; genetics; diet
9. List 5 mechanism that keep venous blood moving toward the right atrium. Blood pressure, clean veins, etc.
10. What’s circulatory shock? List the 5 types. If it gets stopped. Too many to list
11. Name 4 locations on the body where the pulse can be felt. Wrists, inner thighs, jugular veins, heartbeat
12. Explain how the formation of varicose veins is an example of a positive feedback mechanism. Tells one that circulation needs repair.
13. Explain hepatic portal circulation. How is it different from normal circulation, and what advantages are gained from this type of circulation? The hepatic vein drains the abdominal cavity.
14. Explain the differences between normal post-natal circulation and fetal circulation. Base on the environment of the fetus; explain how these differences make fetal circulation more efficient. Fetal circulation gets help from the mom.
END